Integral limits of polar coordinate transformation












2












$begingroup$


In an integration exercise, I need to find the volume of the region bounded by the paraboloid $z=x^2+y^2$ and below by the triangle enclosed by the lines $y=x$, $x=0$, and $x+y=2$ in the $xy$-plane.



Here is the illustration of $xy$-plane and the standard solution:



enter image description here



However, I got some troubles when I try to use the method of polar coordinate transformation to solve it.



$left{
begin{array}{ll}
x=rcdot costheta\
y=rcdot sintheta
end{array}
right.$
, where the Jacobian is $J=r$.



The integral should be



$$
iint_R[(rcdot costheta)^2+(rcdot sintheta)^2]cdot|J|~drdtheta=iint_R r^3~drdtheta
$$



I think the limits of $theta$ should be $frac{pi}{4}simfrac{pi}{2}$, but I'm not sure what are the inner limits. Thanks for helps.










share|cite|improve this question











$endgroup$

















    2












    $begingroup$


    In an integration exercise, I need to find the volume of the region bounded by the paraboloid $z=x^2+y^2$ and below by the triangle enclosed by the lines $y=x$, $x=0$, and $x+y=2$ in the $xy$-plane.



    Here is the illustration of $xy$-plane and the standard solution:



    enter image description here



    However, I got some troubles when I try to use the method of polar coordinate transformation to solve it.



    $left{
    begin{array}{ll}
    x=rcdot costheta\
    y=rcdot sintheta
    end{array}
    right.$
    , where the Jacobian is $J=r$.



    The integral should be



    $$
    iint_R[(rcdot costheta)^2+(rcdot sintheta)^2]cdot|J|~drdtheta=iint_R r^3~drdtheta
    $$



    I think the limits of $theta$ should be $frac{pi}{4}simfrac{pi}{2}$, but I'm not sure what are the inner limits. Thanks for helps.










    share|cite|improve this question











    $endgroup$















      2












      2








      2





      $begingroup$


      In an integration exercise, I need to find the volume of the region bounded by the paraboloid $z=x^2+y^2$ and below by the triangle enclosed by the lines $y=x$, $x=0$, and $x+y=2$ in the $xy$-plane.



      Here is the illustration of $xy$-plane and the standard solution:



      enter image description here



      However, I got some troubles when I try to use the method of polar coordinate transformation to solve it.



      $left{
      begin{array}{ll}
      x=rcdot costheta\
      y=rcdot sintheta
      end{array}
      right.$
      , where the Jacobian is $J=r$.



      The integral should be



      $$
      iint_R[(rcdot costheta)^2+(rcdot sintheta)^2]cdot|J|~drdtheta=iint_R r^3~drdtheta
      $$



      I think the limits of $theta$ should be $frac{pi}{4}simfrac{pi}{2}$, but I'm not sure what are the inner limits. Thanks for helps.










      share|cite|improve this question











      $endgroup$




      In an integration exercise, I need to find the volume of the region bounded by the paraboloid $z=x^2+y^2$ and below by the triangle enclosed by the lines $y=x$, $x=0$, and $x+y=2$ in the $xy$-plane.



      Here is the illustration of $xy$-plane and the standard solution:



      enter image description here



      However, I got some troubles when I try to use the method of polar coordinate transformation to solve it.



      $left{
      begin{array}{ll}
      x=rcdot costheta\
      y=rcdot sintheta
      end{array}
      right.$
      , where the Jacobian is $J=r$.



      The integral should be



      $$
      iint_R[(rcdot costheta)^2+(rcdot sintheta)^2]cdot|J|~drdtheta=iint_R r^3~drdtheta
      $$



      I think the limits of $theta$ should be $frac{pi}{4}simfrac{pi}{2}$, but I'm not sure what are the inner limits. Thanks for helps.







      integration






      share|cite|improve this question















      share|cite|improve this question













      share|cite|improve this question




      share|cite|improve this question








      edited Jan 20 at 10:29







      Darren Tsai

















      asked Jan 20 at 10:19









      Darren TsaiDarren Tsai

      1135




      1135






















          1 Answer
          1






          active

          oldest

          votes


















          1












          $begingroup$

          You're doing it right, well done!



          For the limits, as you said, $theta$ goes from $dfrac{pi}{4}$ and $dfrac{pi}{2}$. For $r$, some drawing helps:enter image description here



          Here $alpha=theta-dfrac{pi}{4}$.



          The limits of $r$ is from $0$ to some length that changes according to $theta$ (or $alpha$). For the highest value, you see in the drawing that $cosalpha=dfrac{sqrt{2}}{r}$ so $r=dfrac{sqrt{2}}{cosalpha}$, and you have $alpha=theta-dfrac{pi}{4}$...






          share|cite|improve this answer











          $endgroup$













          • $begingroup$
            Thank you so much!! But I think $cos~alpha$ should be $frac{sqrt{2}}{r}$ because the length of base-side is $sqrt{2}$.
            $endgroup$
            – Darren Tsai
            Jan 20 at 11:56






          • 1




            $begingroup$
            @DarrenTsai It's a pleasure :) Yes you're right! Sorry.
            $endgroup$
            – Scientifica
            Jan 20 at 12:38













          Your Answer





          StackExchange.ifUsing("editor", function () {
          return StackExchange.using("mathjaxEditing", function () {
          StackExchange.MarkdownEditor.creationCallbacks.add(function (editor, postfix) {
          StackExchange.mathjaxEditing.prepareWmdForMathJax(editor, postfix, [["$", "$"], ["\\(","\\)"]]);
          });
          });
          }, "mathjax-editing");

          StackExchange.ready(function() {
          var channelOptions = {
          tags: "".split(" "),
          id: "69"
          };
          initTagRenderer("".split(" "), "".split(" "), channelOptions);

          StackExchange.using("externalEditor", function() {
          // Have to fire editor after snippets, if snippets enabled
          if (StackExchange.settings.snippets.snippetsEnabled) {
          StackExchange.using("snippets", function() {
          createEditor();
          });
          }
          else {
          createEditor();
          }
          });

          function createEditor() {
          StackExchange.prepareEditor({
          heartbeatType: 'answer',
          autoActivateHeartbeat: false,
          convertImagesToLinks: true,
          noModals: true,
          showLowRepImageUploadWarning: true,
          reputationToPostImages: 10,
          bindNavPrevention: true,
          postfix: "",
          imageUploader: {
          brandingHtml: "Powered by u003ca class="icon-imgur-white" href="https://imgur.com/"u003eu003c/au003e",
          contentPolicyHtml: "User contributions licensed under u003ca href="https://creativecommons.org/licenses/by-sa/3.0/"u003ecc by-sa 3.0 with attribution requiredu003c/au003e u003ca href="https://stackoverflow.com/legal/content-policy"u003e(content policy)u003c/au003e",
          allowUrls: true
          },
          noCode: true, onDemand: true,
          discardSelector: ".discard-answer"
          ,immediatelyShowMarkdownHelp:true
          });


          }
          });














          draft saved

          draft discarded


















          StackExchange.ready(
          function () {
          StackExchange.openid.initPostLogin('.new-post-login', 'https%3a%2f%2fmath.stackexchange.com%2fquestions%2f3080392%2fintegral-limits-of-polar-coordinate-transformation%23new-answer', 'question_page');
          }
          );

          Post as a guest















          Required, but never shown

























          1 Answer
          1






          active

          oldest

          votes








          1 Answer
          1






          active

          oldest

          votes









          active

          oldest

          votes






          active

          oldest

          votes









          1












          $begingroup$

          You're doing it right, well done!



          For the limits, as you said, $theta$ goes from $dfrac{pi}{4}$ and $dfrac{pi}{2}$. For $r$, some drawing helps:enter image description here



          Here $alpha=theta-dfrac{pi}{4}$.



          The limits of $r$ is from $0$ to some length that changes according to $theta$ (or $alpha$). For the highest value, you see in the drawing that $cosalpha=dfrac{sqrt{2}}{r}$ so $r=dfrac{sqrt{2}}{cosalpha}$, and you have $alpha=theta-dfrac{pi}{4}$...






          share|cite|improve this answer











          $endgroup$













          • $begingroup$
            Thank you so much!! But I think $cos~alpha$ should be $frac{sqrt{2}}{r}$ because the length of base-side is $sqrt{2}$.
            $endgroup$
            – Darren Tsai
            Jan 20 at 11:56






          • 1




            $begingroup$
            @DarrenTsai It's a pleasure :) Yes you're right! Sorry.
            $endgroup$
            – Scientifica
            Jan 20 at 12:38


















          1












          $begingroup$

          You're doing it right, well done!



          For the limits, as you said, $theta$ goes from $dfrac{pi}{4}$ and $dfrac{pi}{2}$. For $r$, some drawing helps:enter image description here



          Here $alpha=theta-dfrac{pi}{4}$.



          The limits of $r$ is from $0$ to some length that changes according to $theta$ (or $alpha$). For the highest value, you see in the drawing that $cosalpha=dfrac{sqrt{2}}{r}$ so $r=dfrac{sqrt{2}}{cosalpha}$, and you have $alpha=theta-dfrac{pi}{4}$...






          share|cite|improve this answer











          $endgroup$













          • $begingroup$
            Thank you so much!! But I think $cos~alpha$ should be $frac{sqrt{2}}{r}$ because the length of base-side is $sqrt{2}$.
            $endgroup$
            – Darren Tsai
            Jan 20 at 11:56






          • 1




            $begingroup$
            @DarrenTsai It's a pleasure :) Yes you're right! Sorry.
            $endgroup$
            – Scientifica
            Jan 20 at 12:38
















          1












          1








          1





          $begingroup$

          You're doing it right, well done!



          For the limits, as you said, $theta$ goes from $dfrac{pi}{4}$ and $dfrac{pi}{2}$. For $r$, some drawing helps:enter image description here



          Here $alpha=theta-dfrac{pi}{4}$.



          The limits of $r$ is from $0$ to some length that changes according to $theta$ (or $alpha$). For the highest value, you see in the drawing that $cosalpha=dfrac{sqrt{2}}{r}$ so $r=dfrac{sqrt{2}}{cosalpha}$, and you have $alpha=theta-dfrac{pi}{4}$...






          share|cite|improve this answer











          $endgroup$



          You're doing it right, well done!



          For the limits, as you said, $theta$ goes from $dfrac{pi}{4}$ and $dfrac{pi}{2}$. For $r$, some drawing helps:enter image description here



          Here $alpha=theta-dfrac{pi}{4}$.



          The limits of $r$ is from $0$ to some length that changes according to $theta$ (or $alpha$). For the highest value, you see in the drawing that $cosalpha=dfrac{sqrt{2}}{r}$ so $r=dfrac{sqrt{2}}{cosalpha}$, and you have $alpha=theta-dfrac{pi}{4}$...







          share|cite|improve this answer














          share|cite|improve this answer



          share|cite|improve this answer








          edited Jan 20 at 12:45

























          answered Jan 20 at 11:25









          ScientificaScientifica

          6,78641335




          6,78641335












          • $begingroup$
            Thank you so much!! But I think $cos~alpha$ should be $frac{sqrt{2}}{r}$ because the length of base-side is $sqrt{2}$.
            $endgroup$
            – Darren Tsai
            Jan 20 at 11:56






          • 1




            $begingroup$
            @DarrenTsai It's a pleasure :) Yes you're right! Sorry.
            $endgroup$
            – Scientifica
            Jan 20 at 12:38




















          • $begingroup$
            Thank you so much!! But I think $cos~alpha$ should be $frac{sqrt{2}}{r}$ because the length of base-side is $sqrt{2}$.
            $endgroup$
            – Darren Tsai
            Jan 20 at 11:56






          • 1




            $begingroup$
            @DarrenTsai It's a pleasure :) Yes you're right! Sorry.
            $endgroup$
            – Scientifica
            Jan 20 at 12:38


















          $begingroup$
          Thank you so much!! But I think $cos~alpha$ should be $frac{sqrt{2}}{r}$ because the length of base-side is $sqrt{2}$.
          $endgroup$
          – Darren Tsai
          Jan 20 at 11:56




          $begingroup$
          Thank you so much!! But I think $cos~alpha$ should be $frac{sqrt{2}}{r}$ because the length of base-side is $sqrt{2}$.
          $endgroup$
          – Darren Tsai
          Jan 20 at 11:56




          1




          1




          $begingroup$
          @DarrenTsai It's a pleasure :) Yes you're right! Sorry.
          $endgroup$
          – Scientifica
          Jan 20 at 12:38






          $begingroup$
          @DarrenTsai It's a pleasure :) Yes you're right! Sorry.
          $endgroup$
          – Scientifica
          Jan 20 at 12:38




















          draft saved

          draft discarded




















































          Thanks for contributing an answer to Mathematics Stack Exchange!


          • Please be sure to answer the question. Provide details and share your research!

          But avoid



          • Asking for help, clarification, or responding to other answers.

          • Making statements based on opinion; back them up with references or personal experience.


          Use MathJax to format equations. MathJax reference.


          To learn more, see our tips on writing great answers.




          draft saved


          draft discarded














          StackExchange.ready(
          function () {
          StackExchange.openid.initPostLogin('.new-post-login', 'https%3a%2f%2fmath.stackexchange.com%2fquestions%2f3080392%2fintegral-limits-of-polar-coordinate-transformation%23new-answer', 'question_page');
          }
          );

          Post as a guest















          Required, but never shown





















































          Required, but never shown














          Required, but never shown












          Required, but never shown







          Required, but never shown

































          Required, but never shown














          Required, but never shown












          Required, but never shown







          Required, but never shown







          Popular posts from this blog

          Mario Kart Wii

          What does “Dominus providebit” mean?

          Antonio Litta Visconti Arese